Workshop on Electrolyteand Acid-BaseDisturbances

Organisers AKAgarwal,RKSingal

Editor PraveenAggarwal Governing Body of Association of Physicians of India

PresidentElect President PastPresident AKAgarwal, NewDelhi(2009) SKBichile,(Mumbai 2009) RKSingal,(NewDelhi 2009)

Vice Presidents BRBansode,(Mumbai 2009) PritamGupta,(NewDelhi 2010) AlakaDeshpande,(Mumbai 2011)

Hon.General Secretary Sandhya Kamath,(Mumbai 2010)

Hon.Treasurer Milind Y Nadkar,(Mumbai 2009)

Members ShyamSundar,(Varanasi 2009) RohiniHanda,(NewDelhi 2010) NarinderPalSingh,(NewDelhi 2011) YSatyanarayanRaju,(Hyderabad 2009) AmalKumarBanerjee,(Howrah 2010) MadhukarRai,(Varanasi 2011) JMukhopadhyay,(Howrah 2009) RSajithkumar,(Kottayam 2010) SurendraDaga,(Kolkata 2011) NKSingh,(Dhanbad 2009) AbhayNRai,(Gaya 2010) SArulrhaj,(Tuticorin 2011)

Zonal Members NorthZone–RajeshUpadhyay,(NewDelhi 2011) MidSouthZone–GNarsimulu,(Hyderabad 2011) NorthWestZone–GurpreetSWander,(Ludhiana 2011) SouthZone–AMuruganathan,(Tirupur 2011) CentralZone–SanjivMaheshwari,(Ajmer 2011) MidEastZone–KamleshTewary, Muzaffarpur( 2011) WestZone–AshitBhagwati,(Mumbai 2011) EastZone–SamarKumarBanerjee,(Kolkata 2011)

Ex Officio Member Dean ICP AK Das, Pondicherry

Invited Members Hon.Editor,JAPI APIHouseChairman Editor-in-chief,APITextBook ShashankRJoshi, Mumbai SiddharthNShah, Mumbai YPMunjal, NewDelhi

Co-opted Members Jt.Secretary, President’sPlace ArmedForces, MedicalServices Jt.Secretary(HQ) JMPhadtare, Mumbai(2009) Lt.Gen.SRMehta,(NewDelhi 2007-2008) FalguniParikh, Mumbai

OrganisingSecretary, APICON2008 OrganisingSecretary, APICON2008 NNAsokan, Muvattupuzha NKSoni, Ghaziabad APICON 2009

Workshop on Electrolyte and Acid-Base Disturbances

Date: January 30, 2009

Time: 2.00 pm – 6.00 pm

Convener Praveen Aggarwal, All India Institute of Medical Sciences New Delhi

Faculty l Praveen Aggarwal, New Delhi l Chhavi Sawhney, New Delhi l Sanjeev Bhoi, New Delhi

Electrolyte and Acid-Base Disturbances l APICON 2009 l Jan. 29 - Feb. 1, 2009 1 SCIENTIFIC COMMITTEE, APICON – 2009

A. K. Agarwal Chairman, Scientific Committee S. K. Bichile President, Association of Physicians of India (API) A. K. Das Dean, Indian College of Physicians (ICP) B. B. Thakur Dean-Elect, ICP Sandhya Kamath Hony. General Secretary, API and ICP Shashank R. Joshi Hony. Editor, Journal of Association of Physicians of India (JAPI) N. K. Soni Organising Secretary, APICON - 2009

Members – Scientific Committee R. K. Singal Surendra K. Sharma Samar Banerjee Shyam Sundar

Principal Advisors Y. P. Munjal B. K. Sahay Siddharth N. Shah

Members – Advisory Board S. Arulrhaj T. Kadhiravan J. M. Phadtare Amal Kumar Banerjee O. P. Kalra Prashant Prakash B. R. Bansode Ulka Kamble A. N. Rai Subhash Chandra Umesh Kansra Y. Satyanarayan Raju Anil Chaturvedi Ram Kapoor B. B. Rewari S. Chugh V. N. Kaushal Anita Sharma Naveen Garg Navin Kumar N. K. Singh P. S. Ghalaut Rajat Kumar N. P. Singh Anil Gomber Sanjiv Maheshwari Rita Sood Vinay Gulati Lt. Gen. S. R. Mehta Dinesh Srivastava B. Gupta Vipin Mendiratta B. K. Tripathi B. B. Gupta Adhip Mitra Rajesh Upadhyay Pritam Gupta Alladi Mohan A. K. Vaish Rohini Handa Ashutosh Mohan A. K. Varshney

2 Electrolyte and Acid-Base Disturbances l APICON 2009 l Jan. 29 - Feb. 1, 2009 Contents

1. Fluid and Electrolyte Disturbances 4 Sanjeev Bhoi, Chhavi Sawhney

2. Acid-Base Disturbances 19 Praveen Aggarwal

Electrolyte and Acid-Base Disturbances l APICON 2009 l Jan. 29 - Feb. 1, 2009 3 1. Fluid and Electrolyte Disturbances

Sanjeev Bhoi, Chhavi Sawhney

The sweat, exudates, and transudates can also be considered as specialized portions of the extracellular water, because About a billion years ago, life began-in the sea. The sea when these are lost a severe loss of extracellular water possessed unique properties for the maintenance of life. occurs. For example, the water of the sea is a solvent for the electrolytes and the oxygen that are necessary for life. The relation of the body water to the body weight are as The sea is also a solvent for the carbon dioxide that follows: accumulates during life’s processes. Since the carbon Water Compartment Percentage Volume in dioxide is volatile, it can be easily dissipated from the of Body Liters surface of the sea. In addition, the volume of the sea is Weight (man, 70 kg) so great that it can absorb large amounts of heat, or lose large amounts of heat, with only relatively small changes Plasma water (plus) 4 2.8 in temperature. The volume of the sea is also so great Interstitial water 16 11.2 that significant changes in its composition occur only over a period of hundreds of thousands of years. Finally, Total extracellular its dielectric constant, its surface tension, and other water (plus) = 20 = 14 physical properties are all important in maintaining and Intracellular water 40 28 protecting life. Total average body As a result, the water that surrounds the cells of water in a man = 60 = 42 vertebrates and of humans, namely, the extracellular Total average body water, still has an electrolyte composition similar to what water in a 70 kg woman 50 35 the sea had in pre-recorded times, in spite of all the countless changes in evolution that have occurred. Over These figures represent average values that we shall use the years, the rivers of the world have eroded land and in calculating water and electrolyte requirements in this washed elements into the sea. This has caused the book. electrolytes of the sea to become more concentrated. In The total body water varies. It is related principally to spite of this, there is still a remarkable similarity between the fat content of the body, and to sex. Fat has relatively the proportional composition of electrolytes in seawater less water associated with it; therefore a fat person will and in extracellular water. have relatively less water than a thin person. In addition, The total amount of water in the body can be determined a woman has a lower content of body water than a man. by introducing into the body a known quantity of a The water content of the body also decreases with age. substance that diffuses evenly throughout the The average water content of a man is approximately extracellular water and the cells, and then determining 60%. The average water content of a woman is its concentration. Antipyrine, urea, thiourea, and more approximately 50%. recently “heavy water” (deuterium oxide or tritium oxide) have been used for this purpose. Body water is also composed of inaccessible bone water and transcellular fluids. The Extracellular Water Transcellular fluids include the fluid of organs such as the kidneys, liver, pancreas, skin and the mucous The body water is usually divided into two main membranes of the gastrointestinal and respiratory, tracts; compartments: the extracellular water (extracellular and so on. It is difficult to measure the volume of the fluid) and the intracellular water (intracellular fluid). fluid in these compartments. The extracellular water includes the plasma water and the interstitial water (the fluid in the tissue spaces, lying Body water, including percentage of fat and of lean body between the cells). Gastrointestinal secretions, , tissue, can now be measured using a bioelectrical

4 Electrolyte and Acid-Base Disturbances l APICON 2009 l Jan. 29 - Feb. 1, 2009 impedance analyzer. This is due to the fact that the capillary walls prevent the outward movement of most of the protein from The Electrolytes in the Extracellular Water the plasma. As a result, the sodium concentration in plasma is slightly greater than the sodium Chemists are able to determine the nature and the concentration in the interstitial water. Conversely, the concentration of the electrolytes in both the extracellular chloride concentration in the plasma is slightly less water and the cells. However, the determination of than the chloride concentration in the interstitial water. electrolyte concentration in the cells requires special research techniques. Therefore, physicians must rely on The electrolyte concentrations are given in terms of the changes in electrolyte concentration in the milliequivalents per liter of serum or plasma (mEq/L). extracellular water, particularly in the plasma or serum, in treating patients. Milliequivalents The relative concentration of electrolytes in the The electrolytes of the body are in solution, mostly in extracellular water and in the cells is shown in Table 1. the form of ions. Their concentration can be described Notice that sodium (Na) and chloride (Cl) are the in terms of weight, such as milligrams (mg) per deciliter principal electrolytes in the extracellular water. However, (dL) of blood (mg/dL: a deciliter, or one tenth of a liter, potassium (K) and phosphates (PO4) are the principal is the same as 100 mL). Since the concentrations of the electrolytes in the cells. various electrolytes are small, they are expressed as milliequivalents per liter, mEq/L. Table 1: The Electrolyte concentration of body fluids (mEq/L)* These values are shown in Table 1. Notice that the concentration of the cations is the same as the Solution Extracellular Intracellular concentration of the anions in blood, serum or plasma Fluid Fluid when expressed in terms of milliequivalents. In other Cations words, each liter of blood has 153 mEq of cations (Na, K, Sodium 142 10 Ca, Mg) and 153 mEq of anions (, chloride, sulphate, phosphate, organic acids and protein anions). Potassium 4.5 150 Magnesium 2 40 The relationship between milliequivalents and Calcium 4.5 milligrams Total 153 200 The relationship between milliequivalents and Anions milligrams can be expressed as follows : The weight of a salt in milligrams can be converted into milliequivalents Chloride 102 by dividing its weight in milligrams by its molecular Phosphates 2 120 weight, and multiplying by the valence. Sulphates 4 30 Example: Bicarbonate 27 10 Protein 16 40 1 g of NaCl = 1000/58.5 mEq of NaCl = 17.1 mEq Organic acids 5 Na (at. wt. 23; valence 1) Cl (at. wt. 35.5; valence 1) Na + Cl = 58.5 Total 153 200

*Adapted from Wilson, RF. Critial Care Manual: Applied Physiology Since the electrolyte concentration in blood and serum and Principles of Therapy, ed 2. F.A. Davis, Philadephia, 1992, p are usually expressed in mEq/L, the following formulas 656. can be used to convert mg/dL into mEq/L, or vice versa: It should be noted that the electrolyte concentration mg/dL x 10 x valence of the plasma (or serum) is slightly different from the mEq/L = –––––––––––––––––––– electrolyte concentration in the interstitial portion of Atomic weight the extracellular water. The major difference is that mEq/L x 10 x atomic weight the protein concentration of the plasma is greater that mg/dL = –––––––––––––––––––––––––– that of the interstitial portion of the extracellular water. 10 x valence

Electrolyte and Acid-Base Disturbances l APICON 2009 l Jan. 29 - Feb. 1, 2009 5 The activity constant of ions However, measurement of the (see below) is a simpler way of describing an accumulation of abnormal Measurement of the concentration of an ion or anions. electrolyte in the blood does not necessarily indicate the quantity is completely available for chemical reactions, because a portion may be bound to proteins, The anion gap water, or other ions. A familiar example is the serum The anion gap (or delta) also describes the residual or calcium concentration, which is partly bound to serum unmeasured anions. albumin, and is partly ionized. A similar situation exists with other ions, as Dahms and others have Since the concentration of anions such as phosphates, shown. The ionized and non-ionized percentages of sulphates, and organic acid and protein anions is not these ions in the blood can both be measured. ordinarily measured, the sum of the measured cations However, the clinical importance of this information (Na, K, Ca) will be greater than the sum of the measured is still not known for most ions. anions (HCO3 and Cl). This difference is known as the anion gap (or delta). Cation-anion balance The following simple formula can be used to determine Table 1 shown that the sum of the cations in serum or if an abnormal gap is present: plasma equals 153 mEq/L and the sum of the anions Anion gap = (Na + K) – (HCO3 + Cl). also equals 153 mEq/L. Normally, the anion gap is 16 mEq/L or less. (An anion This electrical equivalence of cations and anions in gap of less than 9 mEq/L is extremely unlikely, and is serum or plasma is maintained regardless of whether probably the result of a laboratory error). the sum of the cations (and anions) is greater or less than 153 mEq/L. It may be greater, for example, when An alternate formula for measuring the anion gap is: water loss occurs. It is often less that 153 mEq/L when Anion gap = Na – (HCO3 + Cl) electrolytes are lost from the body, as in vomiting or When this formula is used, the average, normal value diarrhea. for the anion gap is 12 mEq/L, with a range of 8 to 16 All the cations can be determined clinically, although it mEq/L. is usual to determine only sodium, potassium, and calcium routinely. Chloride is the only anion that is Example: routinely determined in studying electrolyte Na 142 K 4 HCO 27 Cl 103 mEq/L disturbances. The bicarbonate concentration can be 3 calculated from the CO2 content. Under ordinary Anion Gap = (Na + K) – (HCO3 + Cl). conditions, it is usually between 0.6 and 1.8 mEq/L less (142 +4 ) – (27 + 103) than the CO content. However, for purpose of simplicity 2 146 – 130 = 16 mEq/L one can calculate HCO3 concentration by subtracting 1 mEq/L from the CO2 content. This is normal The sum of phosphates, sulphates, organic acid anions, Increased anion gap and proteins is described by the symbol R. 1. Increased unmeasured anions, such as the The significance of anions labeled R accumulation of lactic acid in ; aceto-acetic acid in diabetic ketoacidosis; organic The term R (residual ions) was used to describe or keto acids caused by the ingestion of unmeasured ions, such as plasma proteins and inorganic salicylates, paraldehyde, and ethylene glycol; and organic acid anions in the plasma, which may be organic acids in hyperosmolar hyperglycaemic present normally or may accumulate in some patients non-ketoacidotic diabetic coma; phosphoric, with . sulphuric, and organic acids in azotaemia; formic R is calculated as follow: acid in methanol (methyl alcohol) poisoning; ketogutaric acid in hepatic failure; other anions, R = Na + K + 6.5 (the sum of Ca and Mg) – (HCO + Cl). 3 including high doses of (sodium) penicillin, and Normally, the value of R is 22 mEq/L or less. (sodium) carbenicillin.

6 Electrolyte and Acid-Base Disturbances l APICON 2009 l Jan. 29 - Feb. 1, 2009 2. Decreased unmeasured cations, for example, concentration to be reduced without necessarily potassium, in hypokalaemia (if the alternate formula affecting the sodium or chloride concentration. above is used); calcium in hypocalcaemia; magnesium in hypomagnesaemia. Osmotic Pressure 3. . The plasma proteins give up H A solute is a substance, such as sodium chloride, ions and increase their net negative charge. As a potassium phosphate, glucose, or protein, which can result, the measured anion concentration of the dissolve in a solvent, such as water, to make a solution. plasma is apparently low. In addition, alkalosis is usually associated with a decreased extracellular. The measure of the osmotic pressure of the salt solution is dependent upon the number of particles or ions of (The total anion value must equal the total cation the salt in a given volume of solution. In other words, a value of the plasma. When alkalosis develops, the more concentrated salt solution would have a greater anion value of the proteins increases. However, the osmotic pressure, and a less concentrated salt solution anion value of the proteins is not measured would have a lesser osmotic pressure. Therefore, there is a spurious decrease of the measured anions, and an apparent increase in the Osmotic pressure is measured in terms of osmoles (Osm) anion gap). or milliosmoles (mOsm). An osmole (or a milliosmole) of a substance such as glucose, which does not dissociate 4. Increased unmeasured anions, such as increased into ions, is the same as a mole (or a millimole). phosphate, or sulphate ions, or due to treatment with However, a mole of a salt such as sodium chloride, which intravenous solutions containing lactate, citrate or dissociates almost completely into sodium and chloride acetate ions. ions, equals 2 osmoles. 5. Laboratory errors, with falsely high serum Na or A mole of a salt such as sodium bicarbonate, which falsely low serum Cl or HCO3 values. dissociates into sodium (Na) and bicarbonate (HCO3) ions, also equals 2 osmoles. A mole of a more complex The relation of serum sodium, bicarbonate and chloride salt such as Na HPO , which dissociates into two Na concentrations 2 4 and one HPO4 ions, equals 3 osmoles. The total osmotic There is another relation between the cations and anions pressure of a solution therefore is calculated from the that can be used, namely, the relation between serum sum of all the ions in solution. sodium concentration and the sum of the serum In the human and in animals, it is assumed that the bicarbonate and chloride ions. In Table 1, it will be noted osmotic pressures of the extracellular water and of the that the most important cation is sodium, and that the cells are the same. Water will flow from a region of low most important anions are bicarbonate and chloride. In to a region of higher osmotic pressure. most cases, the following relations between these three ions exist: In the human, the osmotic pressure of the extracellular water and of the cells is 310 mOsm/L. The significance Na = HCO + Cl + 12 mEq/L 3 of this is the following: In the , sodium Example: concentration is 142 mEq/L and the total cation concentration is 155 mEq/L. Practically all the osmotic

Na 142 HCO3 27 Cl 103 mEq/L pressure of the extracellular water is a result of 142 = 27 + 103 + 12 monovalent salts, which ionize into two ions each. Therefore, the osmotic pressure of the extracellular In other words, if the bicarbonate and chloride ion water is twice the cation (or anion) concentration, or 2 concentration in serum or plasma are known, the x 155 = 310 mOsm/L. We can make the following concentration of sodium can be determined from these assumption: Sodium is the chief cation of the values, using the above rule. extracellular water; therefore osmotic pressure should vary proportionately with the sodium concentration of The major exception to this rule occurs in patients with the serum or plasma. In other words, we should be able a metabolic acidosis who show an abnormal anion gap, to use the serum sodium concentration as measure of because the abnormal anions are not ordinarily the osmotic pressure of the extracellular water. measured (see above). This increased anion Unfortunately, this relationship is valid only in normal concentration in the plasma causes the bicarbonate subjects.

Electrolyte and Acid-Base Disturbances l APICON 2009 l Jan. 29 - Feb. 1, 2009 7 The difference between osmotic pressure and oncotic pressure The term “osmotic pressure” should be differentiated from the term “oncotic pressure,” or “colloid osmotic pressure.” The osmotic pressure of a solution varies with the number of molecules in a solution, as was pointed out above. When the molecular weight of a substance is low, there will be more molecules of the substance per unit weight, and the osmotic pressure will be greater. For example, a substance such as mannitol, which has a low molecular weight, will increase the osmotic pressure when given intravenously. However, a substance such as human albumin, or dextran, which has high molecular weight around 80,000, will not increase the osmotic pressure greatly. However, such a substance exerts an oncotic pressure (colloid osmotic pressure) because it is confined by a membrane (the vascular system) to which it is relatively impermeable. Therefore, an infusion of albumin or dextran will greatly increase the oncotic pressure of the blood and will prevent the loss of excessive fluid from the capillaries. However, it will have an almost negligible effect on the Fig. 1: Diagram showing the effect of osmotic pressure. osmotic pressure of the blood. Oncotic pressure is measured in terms of pressure units (mm Hg). skin turgor and dry mucous membranes are poor markers of decreased interstitial fluid. Signs of Factors regulating the osmotic pressure and the volume intravascular volume contraction include decreased of the extracellular water jugular venous pressure, postural hypotension, and We have just pointed out that a primary change of postural tachycardia. Mild degrees of volume depletion osmotic pressure, either in the extracellular water or are often not clinically detectable. Weight loss can help in the cells, is associated with a shift of water into or estimate the magnitude of the volume deficit. Larger out of the extracellular water, and consequently is fluid losses often present as hypovolaemic shock, associated with a change in the volume of the heralded by hypotension, tachycardia, peripheral extracellular water. vasoconstriction, and hypoperfusion-cyanosis, cold and In the maintenance of life, the body has homeostatic or clammy extremities, oliguria, and altered mental status. regulatory mechanisms that help to maintain the osmotic A thorough history and physical examination are pressure and the volume of the extracellular water within generally sufficient to determine the presence and cause physiological limits. The osmotic pressure of the of ECF volume contraction. Laboratory data confirm extracellular water is controlled by the posterior and support the clinical diagnosis. Measurement of the + pituitary antidiuretic harmone and volume of fractional excretion of Na and BUN-creatinine ratio extracellular fluid partly by the adrenal cortical hormone, may provide additional diagnostic information. There . may be a relative elevation in haematocrit (haemoconcentration) and plasma albumin concentration. Hypovolaemia Manifestations Aetiology Symptoms are usually non-specific and secondary to Extracellular fluid (ECF) volume depletion reflects a electrolyte imbalances and tissue hypoperfusion. These deficit in total body Na+ content as a result of renal or include,, fatigue, weakness, muscle cramps, and extrarenal losses that exceed Na+ intake. Renal losses postural dizziness. More severe degrees of volume may be secondary to diuretics (pharmacological or contraction can lead to syncope and coma. Diminished osmotic), interstitial renal disease (Na+ wasting), or

8 Electrolyte and Acid-Base Disturbances l APICON 2009 l Jan. 29 - Feb. 1, 2009 mineralocorticoid deficiency. Excessive renal losses of the post-operative state, and intubated patients in Na+ and water may also occur during the diuretic phase the ICU. Rarely, impaired thirst may be caused by of ATN and after the relief of bilateral urinary tract primary hypodipsia, as a result of damage to the obstruction. Non-renal causes of hypovolaemia include hypothalamic osmoreceptors that control thirst. fluid loss from the GI tract (vomiting, nasogastric Primary hypodipsia may be caused by a variety of suction, fistula drainage, diarrhoea), skin and pathologic changes, including granulomatous respiratory losses, third-space accumulations (burns, disease, vascular occlusion, and tumours. pancreatitis, peritonitis), and haemorrhage. B. Hypernatraemia due to water loss accounts for the majority of cases of hypernatraemia. Because water Treatment is distributed between the ICF and the ECF in a 2:1 The therapeutic goal is to restore normovolaemia with ratio, a given amount of solute-free water loss results fluid similar in composition to that which was lost, as in the same percentage change but, quantitatively, a well as to replace ongoing losses. Mild volume twofold greater absolute reduction in the ICF contraction can usually be corrected via the oral route. compartment than the ECF compartment. More severe cases of hypovolaemia require IV therapy. 1. Non-renal water loss may be due to evaporation Patients with significant haemorrhage, anaemia. or third- from the skin and respiratory tract (insensible spacing may require blood transfusion or colloid- losses) or loss from the GI tract. Insensible losses containing solutions (albumin, dextran). Isotonic or are increased with fever, exercise, heat exposure, normal saline (0.9% NaCI or 154 mEq/L Na+) is the severe burns, and in mechanically ventilated solution of choice in normonatraemic and mildly patients. Diarrhoea is the most common GI hyponatraemic individuals and should also be used cause of hypernatraemia. Specifically, osmotic initially in patients with hypotension or shock. Severe diarrhoeas (induced by lactulose, sorbitol, or hyponatraemia may require hypertonic saline (3.0% malabsorption of carbohydrate) and viral NaCI or 513 mEq/L Na+). Hypokalaemia may be present gastroenteritis result in water loss exceeding that initially or may ensue as a result of increased urinary of Na+ and K+. K+ excretion and should be corrected by adding appropriate amounts of KCI to replacement solutions. 2. Renal water loss is the most common cause of Finally, the appropriate management of hypovolaemia hypernatraemia and results from either osmotic must include correction of the underlying cause. diuresis or diabetes insipidus. The most frequent cause of an osmotic diuresis is hyperglyceamia Hypernatraemia and glycosuria in poorly controlled diabetes mellitus. IV administration of mannitol and Hypernatraemia is defined as a plasma [Na+] of greater increased production of urea (high-protein diet) than 145 mEq/L and presents a state of hyperosmolality. can also result in an osmotic diuresis. Maintenance of osmotic equilibrium in hypernatremia Hypernatraemia secondary to non-osmotic results in ICF volume contraction and cerebral cell urinary water loss is usually caused by (1) shrinkage. Hypernatraemia may be caused by a primary central diabetes insipidus (CDI) characterized Na+ gain or water deficit. The two components of an by impaired secretion or (2) appropriate response to hypernatraemia are increased nephrogenic diabetes insipidus (NDI) that water intake stimulated by thirst and the excretion of results from resistance to the actions of the minimum volume of maximally concentrated urine, vasopressin. The most common cause of CDI is reflecting vasopressin secretion in response to an destruction of the neurohypophysis as a result osmotic stimulus. of trauma, neurosurgery, granulomatous dis- ease, neoplasms, vascular accidents, or infection. Aetiology In many cases, CDI is idiopathic and may occasionally be hereditary. NDI may either be A. Impaired thirst: The degree of hyperosmolality is inherited or acquired. The latter can be further typically mild unless the thirst mechanism is subdivided into disorders associated with renal abnormal or access to water is limited. The latter medullary disease or with impaired vasopressin occurs in infants, the physically handicapped, action. The causes of sporadic NDI are patients with impaired mental status, individuals in numerous and include drugs (especially lithium),

Electrolyte and Acid-Base Disturbances l APICON 2009 l Jan. 29 - Feb. 1, 2009 9 hypercalcaemia, hypokalaemia, and conditions 100 mEq/L). Many causes of hypernatraemia are that impair medullary hypertonicity (e.g., associated with polyuria and a submaximal urine papillary necrosis or osmotic diuresis). osmolality. Calculation of the total daily solute excretion (24-hr urine volume x urine osmolality) C. Hypernatraemia due to Na+ gain occurs infrequently. is helpful in determining the basis of polyuria. To This is most commonly seen in patients with diabetic maintain a steady state, total solute excretion must ketoacidosis (DKA) and an osmotic diuresis (urine equal solute production. As mentioned previously, Na+< 50 mEq/L) treated with isotonic saline. a daily solute excretion in excess of 900 mOsm Inadvertent administration of hypertonic NaCl or defines an osmotic diuresis. This can be confirmed NaHCO or replacing sugar with salt in infant formula 3 by measuring urine glucose and urea. can also lead to hypernatraemia. 2. CDI and NDI generally present with polyuria and D. Transcellular water shift from ECF to ICF occurs in hypotonic urine (urine osmolality <250 mOsm/kg). rare circumstances (e.g., secondary to seizures or The degree of hypernatraemia is usually mild unless rhabdomyolysis). Hypernatraemia is accompanied the patient has an associated thirst abnormality. The by ECF volume contraction with no change in body clinical history, physical examination, and pertinent weight. laboratory data can often rule out causes of acquired NDI. CDI and NDI can be distinguished by Manifestations administering the vasopressin analog DDAVP (10 The major symptoms of hypernatraemia are neurologic microgram intranasally) after careful water and include altered mental status, weakness, restriction. The urine osmolality should increase by neuromuscular irritability, focal neurologic deficits, and at least 50% in CDI and does not change in NDI. occasionally coma or seizures. Patients may also The diagnosis is sometimes difficult due to partial complain of polyuria or thirst. For unknown reasons, defects in vasopressin secretion and action. patients with polydipsia from CDI tend to prefer ice- cold water. The signs and symptoms of volume depletion Treatment are often present in patients with a history of excessive The therapeutic goals are (1) to stop ongoing water loss sweating, diarrhoea, or osmotic diuresis. As with and (2) to correct the water deficit. The ECF volume hyponatraemia, the severity of the clinical should be restored in hypovolaemic patients. The manifestations is related to the acuity and magnitude of quantity of water required to correct the deficit can be + the rise in plasma [Na ]. Chronic hypernatraemia is calculated from the following equation: generally less symptomatic as a result of adaptive mechanisms designed to defend cell volume. (plasma [Na+] – 140) Water deficit = ––––––––––––––––––– x total body water (in liters) 140 Diagnosis 1. The rate of correction: Rapid correction of A complete history and physical examination often hypernatraemia is potentially dangerous due to a provide clues as to the underlying cause of rapid shift of water into brain cells, increasing the hypernatraemia. The history should include a list of risk of seizures or permanent neurologic damage. current and recent medications, and the physical Therefore, the water deficit should be corrected examination is incomplete without a thorough mental slowly over at least 48-72 hours. When calculating status and neurologic assessment. the rate of water replacement, ongoing losses should 1. Assessment of urine volume and osmolality is be taken into account, and the plasma sodium should essential in the evaluation of hyperosmolality. The be lowered by 0.5 mEq/L/hour and by no more than appropriate renal response to hypernatraemia is 12mEq/L over the first 24 hours. The safest route excretion of the minimum volume (500 mL/day) of of administration of water is by mouth or via a maximally concentrated urine (urine osmolality> nasogastric tube. Alternatively, half-isotonic saline 800 mOsm/kg). These findings suggest extrarenal can be given IV. or remote renal water loss or administration of 2. CDl: The appropriate treatment of CDI consists of + + hypertonic Na salt solutions. A primary Na excess administering DDAVP intranasally. can be confirmed by the presence of ECF volume expansion and natriuresis (urine [Na+] usually > 3. NDI: The concentrating defect in NDI may be

10 Electrolyte and Acid-Base Disturbances l APICON 2009 l Jan. 29 - Feb. 1, 2009 reversible by treating the underlying disorder or Oliguric acute and chronic renal failure may be eliminating the offending drug. Symptomatic associated with hyponatraemia if water intake polyuria caused by NDI can be treated with a low exceeds the kidney’s limited ability to excrete Na+ diet and thiazide diuretics. This results in mild equivalent volumes. volume depletion, enhanced proximal reabsorption of salt and water, and decreased delivery to the site 3. Hyponatraemia associated with a normal ECF of action of vasopressin, the collecting duct. NSAlDs volume potentiate vasopressin action’ and thereby increase a. The syndrome of inappropriate antidiuretic urine osmolality and decrease urine volume. hormone secretion (SIADH) is the most common cause of normovolaemic Hyponatraemia hyponatraemia. This disorder is caused by the non-physiologic release of vasopressin This is a condition in which the sodium concentration from the posterior pituitary or an ectopic in the plasma is low (hypo means below in Greek; in source, resulting in impaired renal free water this case, below 135 mmol/L). excretion. Common causes of SIADH include neuropsychiatric disorders, Aetiology pulmonary diseases, and malignant tumours. A. Hyponatraemia with a low plasma osmolality SIADH is characterized by (1) hypotonic hyponatraemia, (2) an inappropriately Most causes of hyponatraemia are associated with concentrated urine (urine osmolality >100 a low plasma osmolality (high ICF volume). In mOsm/kg), (3) euvolaemia, and (4) normal general, hypotonic hyponatraemia is caused either renal, adrenal, and thyroid function. by a primary water gain or Na+ loss. The ECF volume, reflecting total body Na+ content, may be b. Glucocorticoid deficiency and decreased, normal, or increased in hyponatraemia. hypothyroidism may present with hyponatraemia and should not be confused 1. Hyponatraemia associated with ECF volume with SIADH. Although decreased depletion may result from renal or non-renal mineralocorticoids may contribute to the causes of net Na+ loss. A decreased effective hyponatremia seen in Addison’s disease, it arterial volume stimulates thirst. It also is the cortisol deficiency that leads to hyper- stimulates vasopressin release from the posterior secretion of vasopressin directly (co-secreted pituitary gland, which impairs the capacity to with corticotropin-releasing factor) and excrete a dilute urine. Hyponatraemia develops indirectly (secondary to volume depletion). as a consequence of electrolyte-free water The mechanisms by which hypothyroidism retention. Furthermore, certain causes of leads to hyponatraemia include decreased hypovolaemic hyponatraemia (e.g., diuretics or cardiac output and glomerular rate vomiting) may be associated with a large K+ (GFR) and increased vasopressin secretion deficit, resulting in transcellular ion exchange in response to haemodynamic stimuli. (K+ exits and Na+ enters cells), which c. Pharmacologic agents may cause contributes to hyponatraemia. hyponatraemia by one of at least three 2. Hyponatraemia associated with ECF volume mechanisms: (1) stimulation of vasopressin excess is usually a consequence of oedematous release (e.g., nicotine, carbamazepine, states, such as CHF, hepatic cirrhosis, and tricyclic antidepressants, antipsychotic nephrotic syndrome. These disorders all have in agents, antineoplastic drugs, narcotics), (2) common a decreased effective circulating potentiation of antidiuretic action of volume, leading to increased thirst and vasopressin [e.g., chlorpropamide, vasopressin levels. The increase in total body methylxanthines, non-steroidal anti- Na+ is exceeded by the rise in total body water. inflammatory drugs (NSAIDs)], or (3) The degree of hyponatraemia often correlates vasopressin analogs [e.g., oxytocin, with the severity of the underlying condition and desmopressin acetate (DDAVP)]. is therefore an important prognostic factor. d. Physical and emotional stress are often

Electrolyte and Acid-Base Disturbances l APICON 2009 l Jan. 29 - Feb. 1, 2009 11 associated with vasopressin release, possibly artificially lowers the Na+ measured per liter of secondary to nausea and/or hypotension plasma (except when Na+ sensitive glass associated with stress-induced vasovagal electrodes are used). The plasma osmolality and reactions. the Na+ measured per liter of plasma water remain normal. e. Acute hypoxia or hypercapnia also stimulates vasopressin secretion. 2. Hyponatraemia associated with a hyperosmolar state is usually caused by an increase in the f. Psychogenic polydipsia refers to a condition concentration of a solute that is largely restricted of compulsive water consumption that may to the ECF compartment. The resulting osmotic overwhelm the normally large renal gradient leads to water shift from the ICF to the excretory capacity of 12 L/day. These ECF, and hyponatraemia ensues. Hypertonic patients often have psychiatric illnesses and hyponatraemia is usually caused by may be taking medications, such as hyperglycaemia or occasionally IV phenothiazines, that enhance the sensation administration of mannitol. Quantitatively, the of thirst by causing a dry mouth. plasma [Na+] falls by 1.4 mEq/L for every 100 g. Beer potomania is similar to psychogenic mg/dl rise in the plasma glucose concentration. polydipsia but with an associated lower renal excretory capacity of water. Urine can be Manifestations maximally diluted to 50 mOsm/L. The low- solute and low-protein diet seen with The clinical features of acute hyponatraemia are related excessive beer intake may only result in the to osmotic water shift leading to increased ICF volume, generation of 200-250 mOsm/day (600-900 specifically cerebral oedema. Therefore, the symptoms mOsm/day is normal). Thus, only 4-5 L/day are primarily neurologic, and their severity is dependent of urine can be generated. Beer drunk in on the rapidity of onset and absolute decrease in plasma + excess of this capacity results in [Na ]. Patients may be asymptomatic or may complain + hyponatraemia. A similar state, often of nausea and malaise. As, the plasma [Na ] falls, the referred to as the tea-and-toast diet, has been symptoms progress to include headache, lethargy, observed in malnourished elderly patients confusion, and obtundation. Stupor, seizures, and coma + who maintain fluid intake without an do not usually occur unless the plasma [Na ] falls adequate diet. acutely below 120 mEq/L. In chronic hyponatremia, adaptive mechanisms designed to defend cell volume h. Cerebral salt wasting is a controversial and occur and tend to minimize the increase in ICF volume poorly understood syndrome that has been and its symptoms. associated with neurosurgery and CNS trauma. It is purportedly distinguished from Diagnosis SIADH by a negative sodium balance and intravascular volume depletion after a CNS The underlying cause of hyponatraemia can often be injury. The contorversy centers on the tenet ascertained from an accurate history and physical that the underlying hyponatremia is best examination, including an assessment of ECF volume treated with hydration and saline, and not status and effective circulating arterial volume. Three with fluid restriction. laboratory findings often provide useful information and can narrow the differential diagnosis of hyponatraemia: (1) the plasma osmolality, (2) the urine osmolality, and B. Hyponatraemia with a normal or high plasma (3) the urine [Na +] + [CI-]. osmolality 1. Pseudohyponatraemia is hyponatraemia 1. Plasma osmolality: Because ECF tonicity is + associated with a normal plasma osmolality. It determined primarily by the [Na ], most patients occurs as a result of a decrease in the aqueous with hyponatraemia have a decreased plasma phase of plasma. Plasma is 93% water, with the osmolality. If the plasma osmolality is not low, remaining 7% consisting of plasma proteins and pseudohyponatraemia and hypertonic lipids. Because Na+ ions are dissolved in plasma hyponatraemia must be ruled out. water, increasing the non-aqueous phase 2. Urine osmolality and volume: The appropriate renal

12 Electrolyte and Acid-Base Disturbances l APICON 2009 l Jan. 29 - Feb. 1, 2009 response to hypoosmolality is to excrete the cirrhosis tends to reflect the severity of the maximum volume of dilute urine that is, urine underlying disease and is usually asymptomatic. osmolality and specific gravity of less than 100 Treatment should include restriction of Na+ and mOsm/kg and 1.003 respectively. This occurs in water intake, correction of hypokalaemia, and patients with primary polydipsia. If this is not promotion of water loss in excess of Na+. The latter present, it suggests impaired free water excretion may require the use of loop diuretics with due to the action of vasopressin on the kidney. The replacement of a proportion of the urinary Na+ loss secretion of vasopressin may be a physiologic to ensure net free water excretion. Dietary water response to haemodynamic stimuli, or it may be restriction should be less than the urine output. inappropriate in the presence of hyponatraemia and Correction of K+ deficit may raise the plasma [Na+]. euvolaemia. The maximal urine output is a function of the minimum urine osmolality achievable and the 3. The rate of correction of hyponatraemia depends mandatory solute excretion. Metabolism of a normal on the absence or presence of neurologic dysfunction. This, in turn, is related to the rapidity diet generates 600-900 mOsm/day, and the minimum + urine osmolality in humans is approximately 50 of onset and magnitude of the fall in plasma Na . mOsm/kg. Therefore, the maximum daily urine The risks of correcting hyponatremia too rapidly output will be 12 L or more (600/50 = 12). A solute are ECF volume excess and the development of excretion rate of greater than 900 mOsm/day is, by osmotic demyelination or central pontine definition, an osmotic diuresis. A low-protein, low- myelinolysis. This disorder, in its most overt form salt diet may yield as few as 100 mOsm/day, which is characterized by flaccid paralysis, dysarthria, and translates into a maximal urine output of 2 L/day at dysphagia. The diagnosis is occasionally suspected a minimum urine tonicity of 50 mOsm/kg. Moreover, clinically and can be confirmed by appropriate net Na+ loss and ECF volume contraction lead to neuroimaging studies (CT scan or MRI). In vasopressin release, further impairing free water addition to rapid or over-correction of excretion. hyponatraemia, risk factors for osmotic demyelination include hypokalaemia and 3. Urine Na+ concentration: Because Na+ is the major malnutrition, especially secondary to alcoholism. ECF cation and is largely restricted to this compartment, ECF volume contraction represents 4. Acute hyponatraemia tends to present with altered a deficit in total body Na+ content. Therefore, volume mental status or seizures, or both, and requires more depletion in patients with normal underlying renal rapid correction. Severe symptomatic + hyponatraemia should be treated with hypertonic function results in enhanced tubule Na + reabsorption and a urine [Na+] of less than 20 mEq/ saline, and the plasma Na should be raised only by L. The finding of a urine [Na+] of greater than 20 1-2 mEqL/hour and by no more than 8 mEq/L during the first 24 hours. The quantity of Na+ that mEq/L in hypovolaemic hyponatraemia implies + diuretic therapy, hypoaldosteronism, or occasionally, is required to increase the plasma Na concentration vomiting. by a given amount can be estimated by multiplying the desired change in plasma Na+ by the total body Treatment water (e.g., 5 mEq/L x 30 L = 150 mEq = 300 ml 3% NaCI). In asymptomatic patients, the plasma The goals of therapy are threefold: (1) raise the plasma Na+ should be raised by no more than 0.3 mEq/L/ [Na+] (lowering the ICF volume) by restricting water hour and equal to or less than 8 mEq/L over the intake and promoting water loss, (2) replace the Na+ first 24 hours. and K+ deficit(s), and (3) correct the underlying disorder. Mild asymptomatic hyponatraemia is generally of little 5. Water restriction in primary polydipsia and IV saline clinical significance and requires no treatment. therapy in ECF volume-contracted patients may also lead to overly rapid correction of hyponatraemia as 1. ECF volume contraction: Management of a result of vasopressin suppression and a brisk water asymptomatic hyponatraemia should include Na+ diuresis. This can be prevented by administration repletion, generally in the form of saline that is of water or use of a vasopressin analog to slow down isotonic to the patient, to avoid rapid changes in the rate of free water excretion. ICF volume. 6. The hyponatraemia of SIADH can be treated by 2. Oedematous states: Hyponatraemia in CHF and limiting the intake of water or promoting its

Electrolyte and Acid-Base Disturbances l APICON 2009 l Jan. 29 - Feb. 1, 2009 13 Fig. 2: Differential diagnosis of hyponatraemia.

14 Electrolyte and Acid-Base Disturbances l APICON 2009 l Jan. 29 - Feb. 1, 2009 excretion, or both. The standard first-line therapy is – Leukaemia (mechanism uncertain)

water restriction. If this fails or if the patient is l GI losses symptomatic, agents that enhance water excretion – Vomiting or nasogastric suctioning can be tried. Loop diuretics impair the ability to – Diarrhoea excrete concentrated urine and, when combined with Na+ replacement in the form of salt tablets, – Enemas or laxative use can enhance free water excretion. In SIADH, the – Ileal loop urine osmolality is relatively fixed. Therefore, the l Medication effects maximum urine output is a direct function of the – Diuretics (most common cause) solute excretion rate, which can be increased by – Beta-adrenergic agonists dietary modification (high salt, high protein) or by – Steroids administering urea, leading to increased urine output – Theophylline and water excretion. Drugs that interfere with the – Aminoglycosides collecting tubules ability to respond to vasopressin include lithium and demeclocycline. These agents l Transcellular shift are rarely used and should only be considered in – Insulin severe hyponatraemia that is unresponsive to more – Alkalosis

conservative measures. l Malnutrition or decreased dietary intake, parenteral nutrition Hypokalaemia Manifestations Hypokalaemia refers to the condition in which the concentration of potassium in the blood is low. The History prefix hypo- means low (contrast with hyper-, meaning high). Kal refers to kalium, the Neo-Latin for potassium, The history may be vague and -emia means “in the blood”. Common symptoms include the following: Normal serum potassium level is between 3.5 to 5.0 l Palpitations mEq/L. At least 95% of the body’s potassium is found l Skeletal muscle weakness or cramping inside cells, with the remainder in the blood. This l Paralysis, paraesthesias concentration gradient is maintained principally by the l Constipation Na+/K+-ATPase pump. l Nausea or vomiting l Abdominal cramping Pathophysiology l Polyuria, nocturia, or polydipsia l Potassium is essential for many body functions, l Psychosis, delirium, or hallucinations including muscle and nerve activity l Depression l Decreased potassium levels in the extracellular space will cause hyperpolarization of the resting Physical examination membrane potential l Signs of ileus l In certain conditions, this will make cells less l Hypotension excitable. However, in the heart, it causes myocytes l Ventricular arrhythmias to become hyperexcitable. l Cardiac arrest l This delayed repolarization may promote reentrant l Bradycardia or tachycardia arrythmias l Premature atrial or ventricular beats l Hypoventilation, respiratory distress Aetiologies l Respiratory failure l Renal losses l Lethargy or other mental status changes – Renal tubular acidosis l Decreased muscle strength, fasciculations, or tetany – Hyperaldosteronism l Decreased tendon reflexes – Magnesium depletion l Cushingoid appearance (e.g., oedema)

Electrolyte and Acid-Base Disturbances l APICON 2009 l Jan. 29 - Feb. 1, 2009 15 Lab studies Treatment l Serum potassium level <3.5 mEq/L (3.5 mmol/L) Pre-hospital care l BUN and creatinine level l Be attentive to the ABCs. l Glucose, magnesium, calcium, and/or phosphorus l If the patient is severely bradycardic or manifesting level if co-existent electrolyte disturbances are cardiac arrhythmias, appropriate pharmacologic suspected. therapy or cardiac pacing should be considered. l Consider digoxin level if the patient is on a digitalis preparation; hypokalaemia can potentiate digitalis- Emergency department care induced arrhythmias l Patients in whom severe hypokalaemia is suspected l Consider arterial blood gas (ABG). Alkalosis can should be placed on a cardiac monitor; establish cause potassium to shift from extracellular to intravenous access and assess respiratory status. intracellular space l Direct potassium replacement therapy by the symptomatology and the potassium level. Begin Imaging studies therapy after laboratory confirmation of the diagnosis. l CT scan of the adrenal glands is indicated if l Usually, patients who have mild to moderate mineralocorticoid excess is evident (rarely needed hypokalaemia (potassium of 2.5-3.5 mEq/L), are emergently asymptomatic, or have only minor symptoms and need only oral potassium replacement therapy. If Electrocardiography (Figure 3) cardiac arrhythmias or significant symptoms are l T-wave flattening or inverted T waves present, then more aggressive therapy is warranted. This treatment is similar to the treatment of severe l Prominent U wave that appears as QT prolongation hypokalaemia. l ST-segment depression l If the potassium level is less than 2.5 mEq/L, l Ventricular arrhythmias (e.g., premature ventricular intravenous potassium should be given. Admission contractions [PVCs], torsade de pointes, ventricular or observation in emergency department is indicated; fibrillation) replacement therapy takes more than a few hours. l Atrial arrhythmias (e.g., premature atrial l Serum potassium level is difficult to replenish if serum contractions [PACs], atrial fibrillation) magnesium level is also low. Look to replace both.

Fig. 3: ECG showing prominent U waves after T waves and a premature ventricular ectopic (interposed between two QRS complexes).

16 Electrolyte and Acid-Base Disturbances l APICON 2009 l Jan. 29 - Feb. 1, 2009 Medication potassium level. Potassium Potassium depletion sufficient to Complications chloride cause 1 mEq/L drop in serum potassium indicates loss of about 100- l Replacing potassium too quickly can cause a rapid 200 mEq of potassium from total body rise in the blood potassium level, leading to a relative store. In the symptomatic patient with hyperkalaemia with subsequent cardiac severe hypokalaemia, administer up to complications. 40 mEq/h of this IV preparation; l If hypokalaemia is not corrected easily with maintain close follow-up; provide replacement therapy, search for other co-existent continuous ECG monitoring, and metabolic abnormalities (e.g., hypomagnesaemia). check serial potassium levels. Higher Hypokalaemia may be refractory to treatment until dosages may increase risk of cardiac hypomagnesaemia is corrected. complications. l Hypokalaemia can potentiate digitalis toxicity in patients who are taking digoxin Adult dose 10-20 mEq/h IV via peripheral or central line Patient education Pediatric dose 0.5-1 mEq/kg/dose over 1 h; not to l Diet modification is recommended for those patients exceed adult maximum dose who are predisposed to hypokalaemia. Increase intake of bananas, tomatoes, oranges, and peaches Contraindications Hyperkalaemia; renal failure; because they are high in potassium. conditions in which potassium is retained; oliguria or azotaemia; crush Special concerns syndrome; severe haemolytic reactions; l Do not overcorrect potassium in patients with anuria; adrenocortical insufficiency periodic hypokalaemic paralysis. This condition is a Precautions Do not infuse rapidly; high plasma transcellular maldistribution, not a true deficit. concentrations of potassium may cause l Diuretic therapy, diarrhoea, and chronic laxative death due to cardiac depression, abuse are the most common causes of hypokalaemia arrhythmias, or arrest; plasma levels do in elderly patients. not necessarily reflect tissue levels; l In patients with hypokalaemia and diabetic monitor potassium replacement ketoacidosis, part of the serum potassium should be therapy whenever possible by administered as potassium phosphate. continuous or serial ECGs; when concentration >40 mEq/L infused, Hyperkalaemia local pain and phlebitis may occur Hyperkalaemia is defined as potassium level greater than Follow-up 5.5mEq/L. Based on serum potassium concentration, hyperkalaemia can be divided into mild, moderate or Further inpatient care severe. l Continue intravenous replacement of potassium as Mild : 5.5 - 6.0 mEq/L needed. l Continue cardiac monitoring in severe hypokalaemia. Moderate : 6.1 - 7.0 mEq/L l Repeat potassium level measurement every 1-3 hours. Severe : > 7 mEq/L l Identify the aetiology of the hypokalaemia. Etiologies of hyperkalaemia Further outpatient care 1. Intercompartmental shift: l Repeat potassium level in 2-3 days. l Acidosis Inpatient or outpatient medication l Hypertonicity l Consider switching to potassium-sparing diuretic if l Rhabdomyolysis diuretic therapy is needed. l Succinylcholine l Take 40 mEq KCI daily for 2-3 days and repeat the l Digoxin intoxication

Electrolyte and Acid-Base Disturbances l APICON 2009 l Jan. 29 - Feb. 1, 2009 17 l Hyperkalaemic familial periodic paralysis 3. Potassium > 7 mEq/L produces wide p wave, delayed AV conduction, prolonged PR interval 2. Decreased renal potassium excretion 4. Potassium >7.5 mEq/L leads to cessation of atrial l Renal failure contraction and QRS widens resembling a sine wave. l Decreased mineralocorticoid activity Death occurs due to ventricular fibrillation or l Defective tubular secretion ( renal tubular cardiac arrest. acidosis 2 & 4) l Drugs (Spironolactone, ACE inhibitors, Management of hyperkalaemia cyclosporine, NSAIDs) Diagnosis depends upon lab. studies, clinical features 3. Increased potassium intake and ECG changes.

l Salt substitute 1. Calcium gluconate 4. Pseudohyperkalaemia Intravenous 10 ml calcium gluconate 10% over 2-5 minutes. l Haemolysis Onset of action – 5 minutes l Thrombocytosis Duration of action – 30-60 minutes l Leucocytosis Repeat dose in 3-5 minutes l Improper venepuncture technique (ischaemic blood drawn due to prolonged tourniquet Mechanism of action – Potassium antagonism and application) membrane stabilization 2. Insulin-dextrose infusion Clinical manifestations Mechanism of action – Intracellular shift of potassium Skeletal and cardiac muscle Onset of action – 20-30 minutes Skeletal muscle weakness is seen when potassium level Dose – 10 to 20 units regular insuln and 25-50 g is greater than 8 mEq/L. It occurs due to spontaneous glucose sustained depolarization and inactivation of sodium channels on muscle membrane resulting in ascending 3. Intravenous infusion of sodium bicarbonate paralysis. Cardiac manifestations occur when potassium Mechansim of action – Intracellular shift of level is above 7 mEq/L. This results in delayed potassium depolarization of cardiac muscles. Dose – 8.4% solution in children and adults and Various signs and symptoms of hyperkalaemia include 4.2% in infants. Give 1 mEq/Kg slow intravenously the following: as infusion. Precaution – Bicarbonate can bind with calcium; Generalised fatigue therefore, administer bicarbonate 30-60 minutes Weakness after calcium gluconate. Paraesthesias 4. Beta 2 agonist (salbutamol) Paralysis (Decreased motor power, diminished deep Promotes cellular uptake of potassium via cyclic tendon reflexes) GMP receptor cascade. Palpitations, extrasystoles Dose – 5-20 mg mixed with 5 mL of saline via a high Bradycardia, junctional rhythm, heart blocks flow nebulizer over 20 minutes. Hypoventilation (rare) 5. Diuretics Signs of trauma(leading to hyperkalaemia) Furosemide 20-40 mg intravenously Signs of renal failure (oedema, skin changes) Onset of action – 60 minutes Death occurs from cardiac arrest 6. Sodium polystyrene sulfonate (Kayexalate) ECG features Dose – 10-15 g mixed with 100 mL of 20% sorbitol 1. Peaking of T waves and shortened QT interval orally or per rectally 2. Potassium > 6 mEq/L produces wide QRS (Bundle 7. Dialysis is the most effective method when other branch block) methods fail.

18 Electrolyte and Acid-Base Disturbances l APICON 2009 l Jan. 29 - Feb. 1, 2009 2. Acid-Base Disturbances

Praveen Aggarwal

Disturbances in acid-base balance are commonly maintaining the buffering capacity of the encountered in the intensive care units and emergency extracellular fluid (ECF). Other minor buffers in departments. These disorders may be life-threatening ECF include phosphates and proteins. in themselves without regard to the underlying 2. The second physiological buffers are the conditions causing them. Appropriate diagnosis and intracellular proteins, of which, haemoglobin management of these disorders in an acutely ill patient is most important. It can buffer large amounts require accurate and timely interpretation of specific of H+ ions without disturbing the pH. In the data. Physicians must be able to interpret the “numbers” red cells, carbon dioxide combines with water rapidly and accurately. In the present topic, a brief of to form carbonic acid which dissociates to physiological mechanisms by which acid-base balance produce H+ ions that are buffered by is maintained in the body will be discussed. This will be haemoglobin. Other buffers are intracellular followed by various definitions used in acid-base proteins and phosphates. disorders, and detailed description of acid-base disorders and their treatments. Finally, step-by-step approach to 3. Bone contains large amount of bicarbonate and analyze acid-base disturbance will be discussed which can buffer parts of acute acid load. An acid load will be followed by a case to illustrate various principles is associated with uptake of some of the excess of acid-base interpretation. H+ ions by bone. This can occur in exchange for surface Na+ and K+, and by dissolution of bone mineral, resulting in the release of buffer Normal Physiology of Acid-Base Balance compounds such as NaHCO3 and KHCO3 initially and then CaCO and CaHPO into Acids are produced continuously during normal 3 4 metabolism of body. About 20,000 mmol of carbonic extracellular fluid. The role of bone buffers may acid and 80 mEq of non-volatile acids are produced daily. be even greater in the presence of a chronic acid Despite this, the pH of extracellular fluid is maintained retention, such as seen in patients with chronic between 7.36-7.44. The defense against fluctuations in renal failure. pH is provided by three physiological buffer systems, respiratory mechanics and renal mechanics. B. Pulmonary mechanisms The principal volatile acid of metabolism is carbon A. Physiological buffers dioxide which is equivalent to potential carbonic acid. The normal concentration of carbon dioxide The body buffers which are primarily weak acids, in the body is maintained around 1.2 mmol/L by are able to take up or release H+ so that changes in the lungs. At this concentration, the pulmonary the free H+ concentration are minimized. In the excretion equals the metabolic production of carbon body, three type of physiological buffers exist. These dioxide. are as follows: 1. The major physiological buffer is the bicarbonate- C. Renal mechanisms carbonic acid system. Bicarbonate is converted into water and carbon dioxide whenever H+ ions Kidneys reabsorb the filtered bicarbonate and also are added. Carbon dioxide thus liberated is regenerate fresh bicarbonate. Bicarbonate is excreted by the lungs. As the total buffering reabsorbed both in the proximal (75%) and distal capacity of this system is about 15 mEq/L, at segments (25%) by secretion of protons into the normal rate of production of non-volatile acids, tubular fluid. For each molecule of bicarbonate the buffers will be depleted within a period of 15- filtered, one molecule is added to the blood by this 20 days. However, kidneys have the ability to mechanism (Figure 1). regenerate bicarbonate and therefore help in New bicarbonate is regenerated by secretion of

Electrolyte and Acid-Base Disturbances l APICON 2009 l Jan. 29 - Feb. 1, 2009 19 Fig. 3: Formation of NH4 in Renal Tubules.

extracellular volume enhances renal bicarbonate Fig. 1: Renal Reabsorption of Filtered Bicarbonate reabsorption. 3. Aldosterone levels: Hyperaldosteronism stimulates bicarbonate reabsorption by the kidneys and can lead to alkalosis. 4. Body potassium stores: Severe potassium depletion produces increased hydrogen ion secretion and therefore produces alkalosis due to increased bicarbonate reabsorption.

Evaluation of Acid-Base Status Evaluation of acid-base status of a patient is dependent upon changes in the major buffer i.e., the bicarbonate- carbonic acid system. The relationship between bicarbonate and carbonic acid is described by Handerson-Hasselbach equation: Fig. 2: Renal Tubular Titrable Acidity by HPO . 4 Bicarbonate pH = pKa x Log Carbonic acid protons onto urinary buffers. About one-third is titrated to phosphate while the remaining is secreted pKa is known as the dissociation constant and for as ammonium. (Figure 2 and Figure 3). carbonic acid-bicarbonate system, it is 6.1. Carbonic acid can be expressed as dissolved carbon dioxide and equals The net acid excretion by kidneys = urinary NH4 plus α α x pCO2 where represents solubility coefficient and urinary “titrable acid” (H2PO4) minus urinary HCO3. equals 0.031 mmol/L/mmHg of CO2. At a pCO2 of 40 Urinary bicarbonate is almost nil. mmHg, carbonic acid will be 1.2 mmol/L. The arterial

The rate of proton secretion by the kidneys is influenced blood gas analyzers measure the pH and pCO2 and by a number of factors: bicarbonate concentration is calculated using the above- mentioned formula. By modifying the equation stated 1. Carbon dioxide tension: Bicarbonate reabsorption above, a more practical equation can be derived: is directly related to the ECF carbon dioxide tension. + Hypercaponea stimulates and hypocaponea inhibits H in nmol/L = 24 x pCO2/HCO3 renal bicarbonate reabsorption. At a pH of 7.4, the H+ concentration is 40 nmol/L. 2. Extracellular fluid volume: Contraction of It is also important to check the validity of results

20 Electrolyte and Acid-Base Disturbances l APICON 2009 l Jan. 29 - Feb. 1, 2009 obtained by the blood gas analyzer. For this above- to indicate purely metabolic changes. Standard mentioned equation is used to calculate the hydrogen bicarbonate is the bicarbonate concentration in ion concentration and the same is also derived from pH plasma in a completely oxygenated blood sample

(Table 1). If there is discrepancy between the two results, equilibrated with a pCO2 of 40 mmHg at 37°C. A the blood should be analyzed again. standard bicarbonate value below 24 mmol/L indicates metabolic acidosis and above 24 mmol/L Table 1: Relation between pH and Hydrogen ion metabolic alkalosis. This conclusion is based on the concentration assumption that titration of whole-blood with CO2 pH 7.7 7.6 7.5 7.4 7.3 7.2 7.1 7.0 6.9 6.8 produces the same bicarbonate variations in-vitro H+ 20 25 32 40 50 64 80 101 128 160 and in-vivo. Unfortunately, this is not always the (nmol/L) case. Buffer base represents the total equivalent concentration of all anionic (basic) buffer Example: components of the blood, namely hemoglobin, pH = 7.2, pCO = 30, HCO = 22 bicarbonate, plasma proteins and phosphates. It is 2 3 normally 48 mmol/L. 64 = 24 x 30/22 64 ≠ 32 (LAB ERROR) 2. or base deficit: Base excess indicates the deviation of base buffer from its normal value. It can also be defined as the number of mmol of Common Terms used in Evaluation of Acid- strong acid that is needed to adjust the pH to 7.4 Base Status when blood is equilibrated at a pCO2 of 40 mmHg. It is calculated from pH, paCO and haemoglobin. Several terms are used in the evaluation of acid-base 2 disorder. An increase in the amount of buffer base is termed as base excess while a decrease may be referred to Acidaemia: A state when blood pH is below 7.36 as a base deficit or negative base excess. The BE of Alkalaemia: A state when blood pH is above 7.44 oxygenated blood with an Hb of 15 g/dl at a pH of

Acidosis: Any disorder which adds acid or removes base 7.4 and pCO2 of 40mmHg is zero. The BE is strongly from the body influenced by the Hb concentration, which is the Alkalosis: Any disorder which adds alkali or removes main buffer in blood. acid from the body. 3. Standard base excess: It was noted that changes in

(Note: In general, acidosis induces acidaemia and alkalosis pCO2 on HCO3 in vivo differed from that observed induces alkalaemia. However, the difference between these in-vitro and that the discrepancy could be reduced phenomenon becomes important in those patients who have by using a Hb value of 5 g/dL. This empiric value mixed acid-base disturbances in which both acidotic and was presumed to reflect the average concentration alkalotic processes may coexist. In this setting, the net pH of Hb of the fluid space through which bicarbonate may be acadaemic, even though a disorder which induces distributes (between Hb of 15 g/dL in blood to Hb an alkalosis is also present). of 0 g/dl in extracellular fluid). This was called the Respiratory disorders: Disorders where there is standard base excess (SBE). As the SBE was alteration in carbon dioxide concentration initially. independent of pCO2, it was used to define the metabolic component of an acid-base disturbance. Metabolic disorders: Disorders which affect the However, this has not been found useful in many bicarbonate concentration initially. cases and its use is infrequent in analysis of acid- Besides these basic terms, a standard blood gas machine base disorders. measures certain parameters which may be required in 4. Calculated oxygen saturation (%sO2c): It is some patients. These are discussed below. calculated assuming that the oxyhaemoglobin 1. Buffer base and standard bicarbonate: To recognize dissociation curve is not shifted. It will be affected and quantify metabolic disorders, changes in plasma if carboxyhaemoglobin is also present in the sample. bicarbonate are usually evaluated. However, since plasma bicarbonate concentration is also affected Anion Gap by changes in pCO2 i.e., the respiratory disturbances, buffer base and standard bicarbonate have been used An important tool employed in evaluating metabolic

Electrolyte and Acid-Base Disturbances l APICON 2009 l Jan. 29 - Feb. 1, 2009 21 + + acidosis is anion gap. It is calculated as: (Na ) - (HCO3 is a shift of H from the ECF to intracellular fluid while + Cl) and is equal to 8-12 mmol/L. This is because of potassium comes out from the cells into ECF resulting presence of unmeasured anions. Table 2 lists commonly in hyperkalaemia. The reduced pH stimulates the central present anions and cations which are not measured respiratory centre leading to hyperventilation and routinely. Since major anion is albumin, AG must always reduction in carbon dioxide. This in turn reduces the be corrected if patient has significant changes in pH produced by the initial pathology. For hypoalbuminaemia. For every 1 g/dL albumin below 4 each 1 mEq/L decrease in bicarbonate, CO2 reduces by g/dL, add 2.5 to the calculated serum AG. 1.2 mmHg. Winter’s formula for determining compensation in metabolic acidosis is: Table 2: Unmeasured anions and cations pCO = 1.5 [HCO ] + 8 ± 2 Unmeasured Cations: Unmeasured Anions: 2 3 l Total 11 mEq/L l Total 23 mEq/L However, full compensation of pH does not occur and in acute metabolic acidosis, the minimum CO which – Potassium 4 – Sulphates 1 2 can be achieved is 10 mmHg while in chronic acidosis, – Calcium 5 – Phosphates 2 it is 15 mmHg. – Magnesium 2 – Albumin 16 – Lactic acid 1 Pathophysiology – Org. acids 3 Metabolic acidosis can be caused by three mechanisms: Example: 1. Increased production of acids. 2. Decreased excretion of acids by the kidneys A patient has following ABG values: pH 7.20; pCO2 26; + - 3. Loss of alkali from the body HCO3 10; Na 139; Cl 110. His albumin is 2 g/dL.

The anion gap = 139 – (110 + 10) = 19 mmol/L Anion gap Albumin correction: 2.5 x 2 = 5 mmol/L An important tool employed in evaluating metabolic Corrected Anion Gap: 19 + 5 = 24 mmol/L + acidosis is anion gap. It is calculated as: (Na ) - (HCO3 Alterations in the concentration of unmeasured ions can + Cl) and is equal to 8-12 mmol/L. This is because of lead to misestimation of the baseline AG. As an example, presence of unmeasured anions. Increased accumulation both hypoalbuminaemia (reduced unmeasured anions) of these unmeasured anions leads to acidosis with raised and hyperkalaemia (increased unmeasured cations) can anion gap. In some patients of acidosis, there is exchange lower the AG. Thus a patient with one or both of these of bicarbonate with chloride leading to loss of disorders may have a baseline AG of 2 rather than 8 bicarbonate and hyperchlorinaemia. This leads to a meq/L. In this setting, an AG of 14 meq/L, which is normal anion gap with acidosis. only mildly above the normal limit, represents a true It is important to calculate AG in all patients when acid- elevation in the AG of as much as 12 meq/L. base interpretation is being done. An AG > 12 mmol/L can indicate metabolic acidosis while an AG > 20 mmol/ Metabolic Acidosis L always indicates metabolic acidosis. Metabolic acidosis is initiated by a reduction in plasma Diabetic ketoacidosis is typically associated with an bicarbonate concentration. A pH < 7.2 may be life elevated AG. If renal function and volume status is threatening because (i) the enzyme systems become well maintained, however, some or many of the excess unreliable, (ii) electrolyte concentration is altered, (iii) ketone anions may be excreted in the urine as the electrophysiological functions of the body become sodium and potassium salts. The net effect is that the unreliable and this includes impaired cardiac rise in the AG may be much less than expected from contractility and reduced threshold for ventricular the severity of metabolic acidosis. Furthermore, the loss fibrillation, and (iv) the autonomic responses to drugs of these organic anions is equivalent to the loss of are altered. bicarbonate, since metabolism of ketoacid anions results in regeneration of bicarbonate. Thus a normal Compensation in metabolic acidosis AG acidosis is typically seen during the treatment phase of DKA due to the urinary loss of these bicarbonate Because of increased H+ ion concentration in ECF, there precursors.

22 Electrolyte and Acid-Base Disturbances l APICON 2009 l Jan. 29 - Feb. 1, 2009 Aetiology volume of breath. This can produce respiratory fatigue in seriously ill patients. Other non-specific features Important causes of metabolic acidosis are listed in include fatigue, confusion, coma, and cardiovascular Table 3. features due to vasodilatation and reduced cardiac Table 3: Causes of metabolic acidosis contractility which may lead to shock and cardiac failure. Increased Anion Gap Presence of hyperkalaemia (due to shift of intracellular potassium) may mask significant loss of potassium from 1. Severe renal failure body stores. Presence of hypokalaemia in metabolic 2. Increased production of organic acids acidosis indicates severe potassium loss from the body. a. Diabetic ketoacidosis b. Alcoholic ketosis Diagnosis c. Starvation ketosis The first step is to calculate whether the patient has d. Poisonings: Methanol, salicylates, ethylene adequate or whether he has glycol, carbon monoxide, cyanide associated other acid-base disorders. The next step is to e. Increased lactic acid production: calculate anion gap, delta-delta ratio and osmolar gap. Urinary anion gap (UAG) may be of some use in – Cardiorespiratory arrest differential diagnosis. – Convulsions – Shock Delta-Delta ratio or delta ratio – Septicaemia Delta ratio = (Increase in Anion Gap / Decrease in – Liver failure bicarbonate) Normal anion gap (Hyperchlorinaemia) The normal value is between 1 and 1.6. A low delta gap 1. Diarrhea (Loss of alkali) suggests the presence of a concomitant non-gap acidosis, 2. Potassium sparing diuretics (renal tubular whereas a delta gap greater than 1.6 suggests the dysfunction) presence of a concomitant metabolic alkalosis. Another way to remember various causes is given below It might be assumed that there is a 1:1 relationship (Table 4) between the increase in AG (rAG) and the decrease in plasma bicarbonate (rHCO3). However, the r/r ratio Table 4. Causes of Metabolic Acidosis is typically greater than 1 in lactic acidosis, since the High Anion Metabolic Non-anion Gap Metabolic space of distribution of the hydrogen ion and the lactate Acidosis (MUDPILES-R) Acidosis (HARDUPS) anion are different. Most of the lactate anions remain in the extracellular fluid, thereby raising the AG. In l Methanol l Hyperalimentation comparison, more than 50 per cent of the excess l Uraemia l Acetazolamide, hydrogen ions is buffered in the cells and in bone in amphotericin mild metabolic acidosis, a process that does not lower l Diabetic ketoacidosis l Renal tubular acidosis the plasma bicarbonate concentration.Thus, the rise in l Paraldehyde l Diarrhoea anion gap tends to exceed the reduction in the plasma l Iron, INH l Ureteral diversions bicarbonate concentration in lactic acidosis. l Lactic acidosis l Pancreatic fistula In ketoacidosis, the r/r ratio averages about 1:1. In l Ethylene glycol l Saline resuscitation this disorder, the loss of ketoacid anions in the urine l Salicylates lowers the AG without affecting the plasma bicarbonate l Rhabdomyolysis concentration, therefore tending to balance the effect of intracellular buffering of hydrogen. The amount of Clinical features ketoacid anions excreted in ketoacidosis depends on the degree to which glomerular filtration is maintained. Features of metabolic acidosis are non-specific. The Patients with impaired renal function (due to underlying patients usually hyperventilate due to stimulation of diabetic nephropathy or volume depletion) will retain respiratory centre. This is called Kussmaul’s breathing the ketoacid anions and have a relatively high anion that is characterised by increased rate as well as tidal gap in relation to the fall in the plasma bicarbonate

Electrolyte and Acid-Base Disturbances l APICON 2009 l Jan. 29 - Feb. 1, 2009 23 concentration, similar to that in lactic acidosis. (exception is type 2 RTA). UAG is not useful in oliguria and hypovolaemia. Delta gap Urine pH Another term used in the analysis of acid-base disorder is the delta gap. In pure high anion gap metabolic In non-anion metabolic acidosis, urine pH may be useful acidosis, for every 1 mEq/L rise in anion gap, there if the source of loss of fluids is not from intestines. If should be a concomitatnt fall of 1 meq/l in bicarbonate. urine pH > 5.5, type 1 renal tubular acidosis is likely. If Any significant deviation from this relationship suggests urine pH < 5.5, check serum potassium. If serum presence of a mixed acid-base disorder. This can be potassium is low, RTA type 2 is likely; if serum potassium expressed as the delta gap. It equals deviation in anion is high, type 4 RTA is likely. gap from normal minus deviation in bicarbonate from normal. The usual range of delta gap is -6 to +6 and in Treatment case of a pure high anion gap metabolic acidosis, it should be zero. If the calculated delta gap is > 6, this Whenever possible, efforts should be directed at suggests that the bicarbonate has not dropped to the identifying and treating the underlying process which expected extent and thus there is concomitant netabolic gave rise to metabolic acidosis. When metabolic acidosis alkalosis or rarely . A delta gap of results from inorganic acids (i.e., hyperchloraemic or normal anion gap acidosis), HCO is required to treat lesser than – 6 implies a greater loss of bicarbonate than 3 can be accounted for by the metabolic acidosis and the acid-base disturbance. However, when acidosis suggests a concurrent normal anion gap metabolic results from organic acid accumulation (ie, increased acidosis or rarely respiratory alkalosis. However, the anion gap acidosis), as in lactic acidosis, ketoacidosis, or the intoxication syndromes, the role of NaHCO is information gained from delta gap is same as that gained 3 from delta-delta ratio. controversial. Use of intravenous sodium bicarbonate should be restricted to patients with severe acidosis i.e., Osmolar gap bicarbonate < 6 mEq/L. Osmolality of a solution is the number of osmoles of In normal individuals, approximately equal amounts of + solute per kilogram of solvent. Osmolarity of a solution H are buffered by ECF bicarbonate and ICF buffers. is the number of osmoles of solute per litre of solution. Therefore, half of the administered bicarbonate will + The osmolar gap is the difference between the measured accept H from ICF and will be destroyed. Hence, the osmolality and the calculated osmolarity. dose of bicarbonate required = Desired increase in plasma bicarbonate x 40% of body weight (40% is double + Calculated osmolarity: 2 (Na ) + glucose/18 + BUN/ the ECF volume). This volume of 40% of body weight is 2.8 called bicarbonate space. In severe acidosis, bicarbonate Normal osmolar gap is < 10 mOsm/L. It is elevated in space may be large due to intracellular shift of H+ and several conditions including poisoning with ethanol, buffering of H+ by bone and cellular buffers. Therefore, methanol, and ethylene glycol. frequent determinations of acid-base status are required. Half of the calculated bicarbonate should be Urinary anion gap administered first and acid-base estimation is repeated after that before further bicarbonate is administered. In normal circumstances, urine is free of HCO3. Spot urine electrolytes are measured to calculate UAG. NH 4 Example: is the predominant unmeasured cation in urine. It is accompanied by Cl-. A patient weighing 70 kg has following blood gas values: pH = 7.1; pCO = 20 mmHg; HCO = 6 meq/L UAG = Na+ + K+ - Cl- 2 3 + Normal UAG = - 20 to positive The pH should be raised to 7.2. At the pH of 7.2, the H concentration will be 63 nmol/L. Therefore, using the If non-renal source of non-anion gap metabolic acidosis modified Henderson-Hasselbach formula, is present (e.g., diarrhea), there is dramatic increase in 63 = 24 x 20/HCO or HCO = 8 meq/L. NH4 excretion in urine and therefore UAG becomes more 3 3 negative ( < –20 and may even become -50). If patient Presuming bicarbonate space of 70%, the amount of has renal tubular acidosis (RTA), UAG is positive bicarbonate required to increase pH to 7.2 will be:

24 Electrolyte and Acid-Base Disturbances l APICON 2009 l Jan. 29 - Feb. 1, 2009 = Desired increase in bicarbonate x bicarbonate space THAM: Another carbon dioxide-consuming alkalinizing x weight agent is THAM, a commercially available solution of = 2 x 70/100 x 70 = 98 mEq 0.3 N-tromethamine. This sodium-free solution buffers both metabolic acids (THAM + H+ = THAM+) and Initially, half of this i.e., 50 mEq should be administered respiratory acids (THAM + H CO = THAM+ + intravenously. If situation is not very urgent, it may be 2 3 HCO3). Like Carbicarb, THAM limits carbon dioxide administered over a period of 15 minutes. generation and increases both extracellular and Dangers of bicarbonate: There are some dangers of intracellular pH. Nevertheless, THAM has not been bicarbonate therapy, particularly when administered in documented to be clinically more efficacious than excess, and due to these limitations, bicarbonate therapy bicarbonate. In fact, serious side effects, including is at present not recommended in the initial phase of hyperkalemia, hypoglycemia, ventilatory depression, resuscitating patients with cardiorespiratory arrest. local injury in cases of extravasation, and hepatic Various dangers of bicarbonate therapy are: necrosis in neonates, markedly limit its usefulness. 1. Hypokalaemia (due to shift of potassium intracellularly) Metabolic Alkalosis 2. Tetany (due to alkalosis) Metabolic alkalosis is usually initiated by an increased loss of acid from stomach or kidneys. Hypokalaemia 3. Congestive heart failure due to sodium overload occurs because of shift of potassium into intracellular 4. Respiratory alkalosis: Due to reaction of area. administered bicarbonate with H+ ions, carbon dioxide is liberated which diffuses into the brain and Compensation produces acidosis of cerebrospinal fluid. This Alkalosis gives rise to hypoventilation and hence stimulates the respiratory centre leading to retention of CO2 which attenuates the increase in respiratory alkalosis. systemic pH. The compensatory response is that for each

5. Metabolic alkalosis can occur due to excessive 1 mEq/L increase in bicarbonate, CO2 increases by about administration of bicarbonate. Another cause of 0.6 mmHg. Even in the absence of underlying lung metabolic alkalosis is that with the successful disease, secondary hypoventilation is accompanied by treatment of primary disorder, there is rapid some degree of hypoxia. conversion of lactate and ketones into bicarbonate which may lead to alkalosis. Pathophysiology 6. Intracellular acidosis may occur because of Under normal conditions, kidneys excrete excessive intracellular diffusion of liberated carbon dioxide. bicarbonate rapidly so that alkalosis will not be sustained Intracellular acidosis may hamper the functions of unless bicarbonate reabsorption in the kidneys is metabolic enzymes. enhanced or alkali is generated at a great rate. The maintenance of alkalosis is due to three factors: Other options 1. Stimulation of bicarbonate reabsorption due to Carbicarb: It consists of equimolar concentrations of volume depletion when renal conservation of sodium bicarbonate and sodium carbonate. Because sodium takes precedence over correction of carbonate is a stronger base, it is used in preference to alkalosis. Since a large fraction of plasma sodium is bicarbonate for buffering hydrogen ions, generating bound with bicarbonate, complete absorption of bicarbonate rather than carbon dioxide in the process filtered sodium requires reabsorption of bicarbonate + as well. Alkalosis is sustained till volume depletion (CO3 + H = HCO3). In addition, the carbonate ion can react with carbonic acid, thereby consuming carbon exists. On administering sodium chloride, tubular avidity for sodium decreases. Also, chloride is dioxide (CO3 + H2CO3 = 2HCO3). Thus, Carbicarb limits but does not eliminate the generation of carbon available as an alternative to bicarbonate. dioxide. However, the risks of hypervolemia and 2. Another mechanism which can maintain alkalosis hypertonicity are similar to bicarbonate, and neither is hypermineralocorticoidism which stimulates agent prevents progressive reduction in myocardial-cell increased renal H+ secretion as ammonium and pH in animals with ventricular fibrillation. titrable acidity. The patients are not volume-depleted

Electrolyte and Acid-Base Disturbances l APICON 2009 l Jan. 29 - Feb. 1, 2009 25 or chloride-dependent and therefore, do not respond on ECG, and increased susceptibility to dysrrhythmias. to saline infusion. These are either due to hypokalaemia or alkalosis per se. Hypoventilation due to compensation may be 3. The third mechanism of sustaining alkalosis is severe significant in patients with compromised respiration. potassium depletion as renal conservation of potassium takes precedence over H+ resulting in loss of H+ in the urine and hence acidic urine in presence Treatment of alkalosis. Mild-to-moderate alkalosis rarely requires any specific Based on above, various causes of metabolic alkalosis treatment. In presence of severe, saline-responsive are listed in Table 5. alkalosis, infusion of saline and potassium is sufficient to enhance renal excretion of bicarbonate. The ensuing Table 5: Causes of metabolic alkalosis increase in NaHCO3 excretion is due primarily to two Saline Responsive (Urinary Na+ and Cl– < 10 meq/L) events occurring in the collecting tubules – decreased reabsorption and increased secretion of bicarbonate. 1. Loss of chloride-rich, bicarbonate-poor sweat (cystic Repletion of the extracellular volume removes the fibrosis) stimulus to sodium retention, thereby allowing less 2. Diuretics (volume contraction) bicarbonate to be reabsorbed. The increase in distal 3. Use of poorly reabsorbable anions like carbenicillin chloride delivery will raise the chloride concentration + (loss of H ). in the tubular lumen. This will create a more favorable 4. Post-hypercapneic alkalosis gradient for chloride entry into the cells, thereby 5. Gastric alkalosis allowing increased bicarbonate secretion in the cortical Saline Un-responsive (Urinary Na+ and Cl– > 15-20 collecting tubule. meq/L) In saline-resistant cases, specific treatment of primary 1. Severe potassium depletion due to any cause disorder is required. Hypokalaemia should be corrected. 2. Primary aldosteronism and Cushing’s syndrome Spironolactone inhibits secretion of H+ and hence may correct alkalosis. In patients not responding to these In patients with chronic hypercapnoea due to respiratory therapeutic measures, acetazolamide may be used. insufficiency, the plasma bicarbonate is high due to . If the primary condition improves (as In severe cases, intravenous acidifying agents (dilute solutions of arginine hydrochloride, lysine with ventilatory support), pCO2 falls promptly. However, urinary excretion of excessive bicarbonate takes a hydrochloride, and hydrogen chloride) may be required. number of days. In patients who are on low salt diet or Hydrochloric acid administered intravenously as a 0.1 on diuretic and are therefore volume depleted, post- to 0.2 N solution (that is, one containing 100 to 200 hypercapneic alkalosis will persist as kidneys will not mmol of hydrogen per liter) is safe and effective for the be able to excrete bicarbonate from the body. management of severe metabolic alkalosis. The acid can be infused as such or can be added to amino acid and Patients who are having repeated vomiting or are on dextrose solutions containing electrolytes and vitamins continuous gastric aspiration, there is loss of acid leading without causing adverse chemical reactions. Because of to initiation of alkalosis. However, volume and its sclerosing properties, hydrochloric acid must be potassium depletion both tend to maintain alkalosis. administered through a central venous line at an infusion rate of no more than 0.2 mmol per kilogram of body Clinical features weight per hour. However, it can also be administered There are no specific manifestations of metabolic through a peripheral vein if it is added to an amino acid alkalosis. Arteriolar constriction can produce reduced solution and mixed with a fat emulsion. Calculation of cerebral and coronary blood flow. Severe alkalosis may the amount of hydrochloric acid solution to be infused cause apathy, confusion or stupor. The patients may have is based on a bicarbonate space of 50 percent of body increased neuromuscular irritability due to increased weight. Thus, to reduce plasma bicarbonate from 50 to binding of calcium to proteins thereby reducing ionized 40 mmol per liter in a 70-kg patient, the estimated calcium and also because of increased acetylcholine amount of hydrochloric acid required is 10 x 70 x 0.5, release at the neuromuscular junctions. This may or 350 mmol. Precursors of hydrochloric acid, such as produce tetany and seizures. The cardiac manifestations ammonium chloride (20 g per liter, with 374 mmol of are increased QT interval and prominence of U waves hydrogen per liter) and arginine monohydrochloride

26 Electrolyte and Acid-Base Disturbances l APICON 2009 l Jan. 29 - Feb. 1, 2009 (100 g per liter, with 475 mmol of hydrogen per liter), 2. Chronic can substitute for hydrochloric acid, but they entail a. Chronic obstructive lung disease substantial risks and are used less commonly. Both of Disorders of Musculo-skeletal System these preparations are hyperosmotic solutions; to avoid local tissue injury, they must be infused through a central 1. Acute catheter. In addition, ammonium chloride can raise a. Flail chest serum ammonia concentrations in patients with liver b. Tension pneumothorax failure, and arginine monohydrochloride can induce c. Use of aminoglycosides serious hyperkalemia in patients with renal failure, 2. Chronic especially when there is coexisting liver disease. a. Myasthenia gravis b. Poliomyelitis Respiratory Acidosis Respiratory Centre Abnormalities Hypoventilation due to any cause leads to prompt 1. Acute increase in pCO2 leading to respiratory acidosis. Because a. Overdose with opiates and sedatives lungs are so efficient at expiring CO2, any abnormal b. Use of general anesthetic agents increase in carbon dioxide can directly be attributed to c. Cardiac arrest an abnormality affecting this organ system. 2. Chronic Compensation a. Central nervous system lesions In the ECF, there is no buffer to bind CO as bicarbonate 2 Clinical features cannot bind it. In acute respiratory acidosis, the only buffers available are intracellular proteins and It is often difficult to separate manifestations of bicarbonate formed as a result of intracellular buffering respiratory acidosis from those of associated hypoxia diffuses out of cells into ECF increasing its bicarbonate and hypercapnia. Because of hypercapnia, patients may levels and reducing change in pH. In chronic acidosis, have asterixis and they may be obtunded or confused. renal acid secretion is enhanced and bicarbonate is They have increased sweating. Fundus examination may reabsorbed. The response occurs because increased show papilledema. arterial pCO2 increases intracellular pCO2 in proximal tubular cells and this causes increased H+ secretion from Treatment the tubular cells into the tubular lumen. Chloride is The only treatment for severe respiratory acidosis is secreted in place of bicarbonate leading to correction of underlying disorder and use of assisted hypochlorinemic acidosis. Due to these changes, in acute ventilation. Rapid infusion of alkali in the early phase and chronic respiratory acidosis, the bicarbonate of cardiac arrest is not recommended due to various increases by 1 mEq/L and 3.5 mEq/L respectively. complications of bicarbonate therapy.

Pathophysiology Respiratory Alkalosis Any process which decreases ventilation leads to Respiratory acidosis is due to hyperventilation leading respiratory acidosis. Important causes of respiratory to decrease in carbon dioxide. Because carbon dioxide acidosis are listed in Table 6. is excreted by the lungs, the only cause of respiratory Table 6: Causes of respiratory acidosis alkalosis is hyperventilation. Disorders of Gas Exchange Compensation 1. Acute In acute respiratory alkalosis, reduction of CO leads to a. Severe asthma 2 + b. Acute exacerbation of obstructive lung disease release of H from intracellular buffers which decrease plasma bicarbonate levels and hence reduce pH changes. c. Late stages of pulmonary edema In chronic hypocapnia, plasma bicarbonate is further d. Laryngospasm reduced as decreased CO2 inhibits renal reabsorption of e. Foreign body inhalation bicarbonate. In acute and chronic respiratory alkalosis, f. Malfunctioning ventilators the bicarbonate decreases by 2 mEq/L and 5 mEq/L for

Electrolyte and Acid-Base Disturbances l APICON 2009 l Jan. 29 - Feb. 1, 2009 27 every 10 mmHg decrease in CO2. Clinical features The clinical manifestations of respiratory alkalosis Pathophysiology depend on the severity and acuteness of alkalosis. In Hyperventilation can be due to systemic disorders or acute cases, hyperventilation is evident and the patient primary lung disorders (Table 7). complains of paresthesias, numbness, tingling and in some cases, tetany. In severe cases, confusion and coma Table 7: Causes of respiratory alkalosis can occur due to cerebral vasospasm produced by Central Mechanisms hypocapnea. 1. Anxiety 2. Fever Treatment 3. CNS infections The mainstay of treatment is control of underlying 4. Cerebrovascular accident disease. In acute hyperventilation syndrome due to 5. Metabolic encephalopathy anxiety, with syncope and tetany, sedation and 6. Septicemia rebreathing are required. 7. Salicylate poisoning 8. Cirrhosis of liver Primary and Compensatory Changes in Acid- Pulmonary Mechanisms Base Disorders 1. Interstitial lung disease Table 8 and 9 summarize the primary and 2. Asthma compensatory changes in various disorders of acid-base 3. Pneumonia imbalance. 4. Congestive heart failure 5. Pulmonary embolism

Table 8: Primary and compensatory responses in various acid-base disorders Disorder pH Primary response Compensatory response

Metabolic acidosis Decreased Decreased HCO3 Decreased pCO2

Metabolic alkalosis Increased Increased HCO3 Increased pCO2

Respiratory acidosis Decreased Increased CO2 Increased HCO3

Respiratory alkalosis Increased Decreased CO2 Decreased HCO3

Table 9: Compensatory responses in acid-base disorders Disorder Compensation Limit of compensation

Metabolic acidosis For every 1 mEq/L fall in HCO3, pCO2 does not fall below

pCO2 falls by 1.2 mmHg 10 mmHg

Metabolic alkalosis For every 1 mEq/L rise in HCO3, pCO2 does not rise above

pCO2 rises by 0.6 mmHg 55 mmHg

Acute respiratory For every 10 mmHg rise in pCO2, HCO3 does not rise above acidosis HCO3 rises by 1 mEq/L 30 mEq/L

Chronic respiratory For every 10 mmHg rise in pCO2, HCO3 does not rise above acidosis HCO3 rises by 3.5 mEq/L 45 meq/L

Acute respiratory For every 10 mmHg fall in pCO2, HCO3 does not fall below alkalosis HCO3 falls by 2 mEq/L 18 mEq/L

Chronic respiratory For every 10 mmHg fall in pCO2, HCO3 does not fall below alkalosis HCO3 falls by 5 mEq/L 12 mEq/L

28 Electrolyte and Acid-Base Disturbances l APICON 2009 l Jan. 29 - Feb. 1, 2009 Mixed Acid-Base Disorders respiratory alkalosis. Presence of sepsis may point towards mixed metabolic acidosis and respiratory Disorders of mixed acid-base imbalance are quite alkalosis. Further, acid-base abnormalities should be common in sick patients as these patients may be on suspected in patients who are critically ill, are multiple drugs and ventilatory support. Important experiencing extreme dyspnoea, or have abnormal causes of mixed acid-base disturbances are shown in mental status. Table 10. Table 10: Causes of mixed acid-base disorders Step 2: Acidemia or alkalemia. Whether values suggest academia (pH < 7.38) or alkalemia (pH > 7.42). Respiratory Acidosis with Metabolic Acidosis 1. Cardiac arrest Step 3: Identify primary disorder. Look at pH, pCO2

2. Severe pulmonary edema and HCO3. If the pH is below 7.38, and both pCO2 and 3. Drug poisoning with severe respiratory depression bicarbonate are low, metabolic acidosis is present. If pH

is below 7.38 and both pCO2 and bicarbonate are high, Respiratory Alkalosis with Metabolic Alkalosis it indicates presence of respiratory acidosis. 1. Hepatic failure with diuretic use If the pH is higher than 7.42, alkalosis is present. Again 2. Patients on ventilator and nasogastric suction measure pCO2 and bicarbonate. If both carbon dioxide Mixed Acute and Chronic Respiratory Acidosis and bicarbonate are high, metabolic alkalosis is present. 1. Chronic obstructive airway disease with If both carbon dioxide and bicarbonate are low, superimposed infection respiratory alkalosis is present. Respiratory Alkalosis with Metabolic Acidosis Step 4: Calculate compensatory response. The formulas 1. Septic shock are listed in table 9. In acute metabolic acidosis for 2. Salicylate overdose example, Winter’s Formula can be used to calculate whether the respiratory compensation is appropriate. 3. Renal failure with sepsis In metabolic acidosis, pCO2 should be equal to 1.5 times

Respiratory Acidosis with Metabolic Alkalosis bicarbonate levels plus 8 ± 2. If the patient’s pCO2 is 1. COAD with diuretic use higher, the patient has a superimposed respiratory acidosis. If the pCO2 is below this range, the patient has Metabolic Acidosis with Metabolic Alkalosis a superimposed respiratory alkalosis. 1. Diarrhea and vomiting 2. Metabolic acidosis with excessive bicarbonate Step 5: Calculate anion gap. Presence of high anion gap therapy indicates high anion-gap metabolic acidosis. Step 6. Calculate delta-delta ratio. Summary Delta–Delta = Measured AG – Normal AG Fig. 4 Shows approach to interpretation of acid-base disorders. Normal [HCO3] – Measured [HCO3] Delta-delta ratio < 1 indicates associated non-anion gap Step-by-Step Approach to Interpret metabolic acidosis. A delta-delta ration > 1.6 indicates associated metabolic alkalosis. Step 1: Comprehensive history and examination. Comprehensive history taking and physical examination Step 7. Calculate osmolar gap. (if metabolic acidosis is can often give clues as to the underlying acid-base detected). A high osmolar gap indicates presence of one disorder. For example, patients with hypotension, severe of the alcohols. diarrhea or renal failure are likely to have metabolic acidosis. Patients with recurrent vomiting, nasogastric Step 8. Formulate differential diagnosis. What is the aspiration or on diuretics may have metabolic alkalosis. cause of acid-base disorder? Similarly, patients with chronic obstructive lung disease and flail chest are likely to have respiratory acidosis while those with pulmonary embolism may have

Electrolyte and Acid-Base Disturbances l APICON 2009 l Jan. 29 - Feb. 1, 2009 29 Fig. 4: Interpretation of Arterial Blood Gas Report.

Lastly, one case scnario will be discussed to give step-by-step approach to acid-base interpretation.

Practice Case l 60 year male presents to the ED with rapid breathing lll Step 1: Is there an acidemia or alkalemia? and less responsive than usual. No other history is Acidemia available ABG: pH 7.31 Chem: Na+ 123 ABG: pH 7.31 Chem: Na+ 123 PCO 10 K+ 5.0 PCO 10 K+ 5.0 2 2 HCO 5 Cl 99 – 3 HCO3 5 Cl 99 HCO3 5 HCO3 5

30 Electrolyte and Acid-Base Disturbances l APICON 2009 l Jan. 29 - Feb. 1, 2009 lll Step 2. Is the primary process metabolic or lll Step 6: If high anion gap acidosis, calculate delta- respiratory? delta ratio PCO = 10 should drive pH ↑↑↑ 2 Delta–Delta = Measured AG – Normal AG ↓↓↓ HCO3 = 5 should drive pH Normal [HCO3] – Measured [HCO3] ABG: pH 7.31 Chem: Na+ 123 Delta-Delta = (19 – 10)/(24 – 5) = 0.47 PCO 10 K+ 5.0 2 A Non-anion Gap Metabolic Acidosis also

HCO3 5 Cl 99 ABG: pH 7.31 Chem: Na+ 123 HCO3 5 + PCO2 10 K 5.0

HCO3 5 Cl 99 HCO 5 lll Step 3: Choose appropriate compensation formula: 3

In Metabolic acidosis, pCO2 = 1.5 [HCO3] + 8 ± 2 + ABG: pH 7.31 Chem: Na 123 lll Step 7: If high anion gap acidosis, calculate Osmolar + gap PCO2 10 K 5.0 HCO 5 Cl 99 3 Osmolar gap = Measured Osmolality –

HCO3 5 Calculated Osmolarity Calculated Osmolarity = 2 (Na) + (Glucose/18) + (BUN/2.8) lll Step 4: Is the respiratory compensation adequate? ABG: pH 7.31 Chem: Na+ 123 + Metabolic acidosis pCO2 = 1.5 [HCO3] + 8 ± 2 PCO2 10 K 5.0 HCO 5 Cl 99 Expected pCO2 = 1.5 (5) +8 ± 2 = [13.5 – 17.5] 3 HCO 5 PCO2 = 10, additional respiratory alkalosis 3 ABG: pH 7.31 Chem: Na+ 123 + PCO2 10 K 5.0

HCO3 5 Cl 99

HCO3 5

lll Step 5: Calculate the anion gap:

+ – Anion gap = [Na ] – [(Cl ) + (HCO3] Anion gap = 123 – (99 + 5) = 19 Anion Gap Metabolic Acidosis ABG: pH 7.31 Chem: Na+ 123 + PCO2 10 K 5.0

HCO3 5 Cl 99

HCO3 5

Electrolyte and Acid-Base Disturbances l APICON 2009 l Jan. 29 - Feb. 1, 2009 31

Faculty Council of Indian College of Physicians

Chairman Dean SKBichile, Mumbai(2009) AKDas, Pondicherry( 2010)

DeanElect ViceDeans PastDean BBThakur, Muzaffarpur( 2010) SKGoyal,(Kota 2010) YPMunjal,(NewDelhi 2010) OPKalra,(NewDelhi 2010) AnilChaturvedi,(NewDelhi 2010)

Hon.General Secretary Sandhya Kamath,(Mumbai 2010)

Jt.Secretary(HQ) Jt.Secretary(Dean’splace) Hon.Treasurer FalguniSParikh,(Mumbai 2010) TKDutta, Pondicherry( 2010) MilindYNadkar,(Mumbai 2009)

Elected Members SChugh,(Delhi 2009) SubhashChandra,(NewDelhi 2009) MPShrivastava,(NewDelhi 2009) BNJha, Muzaffarpur( 2009) HMLal,(Hyderabad 2009) SChandrasekharan,(Chennai 2010) VinayGoel,(NewDelhi 2010) RKGoyal,(Ajmer 2010) RajatKumar,(NewDelhi 2010) MPSingh,(Ranchi 2010)

Ex-Officio Members PresidentElect Hon.Editor, JAPI AKAgarwal ShashankR.Joshi NewDelhi( 2009) Mumbai( 2009)